Đến nội dung

LangTu Mua Bui nội dung

Có 43 mục bởi LangTu Mua Bui (Tìm giới hạn từ 29-04-2020)



Sắp theo                Sắp xếp  

#601709 Đề thi Olympic toán sinh viên ĐH BK HN 2013

Đã gửi bởi LangTu Mua Bui on 05-12-2015 - 11:59 trong Thảo luận về các kì thi, các kì kiểm tra Toán sinh viên

 Cách khác cho câu 3

$u(x)\leq 1+\int_{0}^{x}\frac{\varphi '(t)u(t)dt}{\varphi (t)}$

Dễ thấy $ u(0) \leq 1$
 
$\Leftrightarrow u(x)-\varphi (x)  \leq  \int_{0}^{x} \frac{\varphi '(t)u(t)dt}{\varphi (t)}-\int_{0}^{x}\varphi '(t)dt=\int_{0}^{x}\left (\varphi '(t)( \frac{u(t)-\varphi (t)}{\varphi (t)})  \right ) dt $

Do $\varphi (t)$ đồng biến và $\varphi(0)=1 \Rightarrow \varphi (t)\geq 1 \forall t\in [0;\infty) $

$\Rightarrow u(x)-\varphi (x) \leq  \int_{0}^{x}\left (\varphi '(t)(u(x)-1)  \right ) dt=u(x)-\varphi (x)-\int_{0}^{x}\varphi '(x) $

$\Rightarrow \varphi(x)<1-\int_{0}^{x}u'(t)\varphi (t)dt ;\varphi(x)\geq 1 \forall x\in [0;\infty] \Rightarrow  u'(t)<0 $

Xét hàm số $g(x)=u(x)-\varphi (x) $

$g'(x)=u'(x)-\varphi' (x) <0 ;g(0)=u(0)-\varphi (0)<0 \Rightarrow g(x)<0 \forall x\in [0;\infty]$



#601708 Đề thi Olympic toán sinh viên ĐH BK HN 2013

Đã gửi bởi LangTu Mua Bui on 05-12-2015 - 11:58 trong Thảo luận về các kì thi, các kì kiểm tra Toán sinh viên

Câu 5 
$\left\{ \begin{array}{l} f'\left( x \right) > 0\\ f\left( {f'\left( x \right)} \right) = - f(x) \end{array} \right.$

Ta có $f(f'(x))=-f(x)$

Thay $ x=f'(x) \Rightarrow f(f'(f'(x))))=f(x)$ Do $f'(x)>0 \Rightarrow f'(f'(x))=x  (1)$

$ \Rightarrow  f(f'(x))=-f(x)$

Do hàm khả vi cấp 2 nên Ta  đạo hàm 2 vế theo x $ \Rightarrow f''(x)f'(f'(x))=-f'(x)(2)$
 
Từ $ (1)(2)f''(x).f'(f'(x))\Leftrightarrow f''(x)x+f'(x)=0$

$ \Rightarrow f(x)=a\ln{x}+b$ Thay ngược lại đề bài $\Rightarrow  a=b=1 \Rightarrow f(x)=\ln{x} $



#601684 Đề thi Olympic toán sinh viên 2013 ĐHSP HCM môn giải tích

Đã gửi bởi LangTu Mua Bui on 05-12-2015 - 01:33 trong Thảo luận về các kì thi, các kì kiểm tra Toán sinh viên

Câu 4 Ta sử dụng Larange để giải bài này

Xét $g(x)=\ln{\sin{f(x)}}$


Theo đl Larange ta có 
$g'(c)=\frac{g(b)-g(a)}{b-a}=\frac{\ln{\sin{f(b)}}-\ln{\sin{f(a)}}}{b-a}$với $g'(c)=f'(c)\cot{f(c)}$(1)

$\frac{1}{a-c}<0<\frac{1}{b-a}<\frac{1}{b-c}$(2) 

Mặt khác $\ln{\sin{f(b)}}-\ln{\sin{f(a)}}<2$ (3)

Từ (1)(2)(3) Ta được đpcm



#601542 Đề thi Olympic sinh viên năm 2013 trường ĐHKHTN - ĐHQG Hà Nội

Đã gửi bởi LangTu Mua Bui on 04-12-2015 - 09:31 trong Thảo luận về các kì thi, các kì kiểm tra Toán sinh viên

Bài 4. Cho $f:\left( {0, + \infty } \right) \to $ thỏa mãn các điều kiện sau
i. $\mathop {\lim }\limits_{x \to + \infty } \left( {f\left( {x + 1} \right) - f(x)} \right) = + \infty $.
ii. $f$ bị chặn trên mọi khoảng con hữu hạn của $\left( {0, + \infty } \right)$.
Chứng minh rằng $\mathop {\lim }\limits_{x \to + \infty } \frac{{f(x)}}{x} = + \infty $.

Không rảnh nhưng thấy bài toán hay nên post lời giải lên… :lol:

Giải.

Theo giả thiết $f$ bị chặn trên $\left( {0, + \infty } \right)$. Lại có $\mathop {\lim }\limits_{x \to + \infty } \left( {f\left( {x + 1} \right) - f(x)} \right) = + \infty $ nên với mọi $M > 0,\exists {x_0} > 0$ sao cho với mọi $x \ge {x_0}$ ta có $f\left( {x + 1} \right) - f(x) > M$.
Sử dụng liên tiếp bất đẳng thức trên ta suy ra

$M + f(x) < f\left( {x + 1} \right) < f\left( {x + 2} \right) - M < ... < f\left( {x + n} \right) - \left( {n - 1} \right)M$.

Suy ra

$f\left( {x + n} \right) > nM + f(x)$ với mọi $x \ge {x_0}$,

nói riêng ta có $f\left( {{x_0} + n} \right) > nM + f({x_0}) \Rightarrow \frac{{f\left( {{x_0} + n} \right)}}{n} > M + \frac{{f({x_0})}}{n}$.
Do $f$ bị chặn nên với $n$ đủ lớn suy ra $\frac{{f\left( {{x_0} + n} \right)}}{n} > M$ tức $f\left( {{x_0} + n} \right) > 0$. Khi đó với $n$ đủ lớn luôn tồn tại ${x_0} \le n$. Do đó

$\frac{{f\left( {{x_0} + n} \right)}}{{{x_0} + n}} \ge \frac{{f\left( {{x_0} + n} \right)}}{{2n}} > \frac{M}{2} + \frac{{f({x_0})}}{{2n}}$.

Suy ra với mọi $x > {x_0} + 2n$ thì $\frac{{f(x)}}{x} > \frac{M}{2}$ với mọi $M > 0$. Do đó $\mathop {\lim }\limits_{x \to + \infty } \frac{{f(x)}}{x} = + \infty $.
Bài toán được chứng minh.

 

Bài này sao không sử dụng ĐL stolez luôn với $a_{n}=f(n)$ 




#601536 Đề thi Olympic sinh viên năm 2013 trường ĐHKHTN - ĐHQG Hà Nội

Đã gửi bởi LangTu Mua Bui on 04-12-2015 - 09:14 trong Thảo luận về các kì thi, các kì kiểm tra Toán sinh viên

Tối nay ngồi post lời giải vậy :D

Giả sử $g(b) \neq 0$, không mất tính tổng quát, giả sử $g(b)>0$, khi đó

$$\lim_{x \to +\infty} \varphi' (x)=g(b) $$

$$\Leftrightarrow \forall \epsilon>0, \exists x_0, \forall x \ge x_0 \rightarrow |\varphi'(x)-g(b)|<\epsilon $$

$$\Rightarrow \varphi'(x)>g(b)-\epsilon$$

Chọn $\epsilon$ đủ nhỏ sao cho $g(b)-\epsilon>0$, suy ra $\int_{x_0}^x \varphi' (t)dt>\int_{x_0}^x (g(b)-\epsilon)dt $

$$\Leftrightarrow \varphi(x)>(g(b)-\epsilon)(x-x_0)+\varphi(x_0)$$

$$\Rightarrow \lim_{x \to +\infty} \varphi(x)=+\infty$$

Mâu thuẫn.

Vậy $g(b)=0$

g(b)=0,0000...1 thì sao 




#598960 ĐỀ THI GIỮA KÌ MÔN GIẢI TÍCH 20151 (ĐHBKHN)

Đã gửi bởi LangTu Mua Bui on 18-11-2015 - 16:59 trong Giải tích

Khai triển taylor tại $x=2015  \Rightarrow P(x)=P(2015)+\sum_{k=1}^{n}\frac{P^{k}(2015)(x-2015)^{k}}{k!}$

$P(x)=1+\sum_{k=1}^{n}(-1^{k})(x-2015)^{k}=1+\sum_{k=1}^{2015}(2015-x)^{k}$

$\Rightarrow P(2014)=1+2015=2016$




#601682 Đề thi Olympic toán học sinh viên 2012 Đại Học BK Hà Nội

Đã gửi bởi LangTu Mua Bui on 05-12-2015 - 01:10 trong Thảo luận về các kì thi, các kì kiểm tra Toán sinh viên

Câu 1
$x_n=\underbrace{\sqrt[3]{6+\sqrt[3]{6+...+\sqrt[3]{6}}}} $

Ta có $2=\sqrt[3]{8}=\sqrt[3]{6+2}=\sqrt[3]{6+\sqrt[3]{8}}=\sqrt[3]{6+\sqrt[3]{6+\sqrt[3]{6+...}}}$

$\Rightarrow \lim_{n\to\infty}6^n(2-x^{n})=0$



#601683 Đề thi Olympic toán học sinh viên 2012 Đại Học BK Hà Nội

Đã gửi bởi LangTu Mua Bui on 05-12-2015 - 01:21 trong Thảo luận về các kì thi, các kì kiểm tra Toán sinh viên

Xét $g(x)=\int_{0}^{x}f(t)dt$ Do f(x) khả vi 2 lần nên g(x) khả vi 3 lần 

Ta có$ g'(x)=f(x) ..$
 
Khai triển maclaurank $\Rightarrow g(x)=g(0)+g(0)x+\frac{g'(0)x^{2}}{3}+\frac{g''(0+a(x-x_{0}))x^{3}}{6}$ (0<a<1)

Thay x=1 ta được $g(1)=\int_{0}^{1}=f(0)+\frac{f'(0)}{2}+\frac{f''(\theta (x))}{6}$ với $c=0<\theta (x)<1$ Ta đpcm 
 



#601477 Đề thi chọn đội tuyển Olympic toán sinh viên 2013 học viện tài chính...

Đã gửi bởi LangTu Mua Bui on 03-12-2015 - 20:56 trong Thảo luận về các kì thi, các kì kiểm tra Toán sinh viên

Câu 5 
$\lim_{x\to\infty}g(x)=\lim_{x\to\infty}\frac{e^{x}g(x)}{e^{x}}(Lopitan)$

$ =\lim_{x\to\infty}\frac{e^{x}(g(x)+g'(x))}{e^{x}} \Rightarrow \lim_{x\to\infty}g'(x)=0 $

$\Leftrightarrow 0=\lim_{x\to\infty}f(g(x)) $ Do f và g là 2 hàm liên tục nên $\lim_{x\to\infty}f(g(x)) $

$=f(\lim_{x\to\infty}g(x))=f(c) \Rightarrow f(c)=0$



#598981 Tính tổng S= $\sum\limits_{k=1}^{2006} f(k...

Đã gửi bởi LangTu Mua Bui on 18-11-2015 - 19:53 trong Các dạng toán THPT khác


$f(n+1)=n(-1)^{n+1}-2f(n) \Leftrightarrow (-1)^{n+1}f(n+1)=n+2(-1^{n})f(n)$

$\Leftrightarrow (-1)^{n+1}f(n+1)+(n+1)=2(n+(-1^{n})f(n))+1 $

$u_{n}=(-1)^{n}f(n)+n \Rightarrow u_{n}=A2^{n}+B$

$u_{1}=-f(1)+1=2A+B;u_{2005}=-f(2005)+2005=A2^{2005}+B$
$\Rightarrow \left\{\begin{matrix} -f(1)+1=2A+B \\-f(2005)+2005=A.2^{2005}+B \\ \end{matrix}\right.$

Tìm được A và B $\Rightarrow f(n)=(-1)^{n}\left ( A.2^{n} +B-n \right )$ Từ đây dễ dàng tính được tổng 



#601388 Tuyển tập một số bài toán Olympic SV Giải tích

Đã gửi bởi LangTu Mua Bui on 03-12-2015 - 14:52 trong Thảo luận về các kì thi, các kì kiểm tra Toán sinh viên

 Câu :3

Đặt 
$a_{n}=\int_{0}^{1}f^{n}(x)dx $

$\Rightarrow \lim_{n \to \infty}\sqrt[n]{a_{n}}=\lim_{n \to \infty}e^{\frac{\ln{a_{n}}}{n}}=\lim_{n \to \infty}e^{\ln{\frac{a_{n+1}}{a_{n}}}}=\lim_{n \to \infty}\frac{a_{n+1}}{a_{n}} $

 

Như ta đã biết $ \int_{0}^{1}f^{n}(x)dx=\sum_{i=0}^{i=n}f^{n}(\frac{i}{n})$

 $\Rightarrow \frac{\int_{0}^{1}f^{n+1}(x)dx}{\int_{0}^{1}f^{n}(x)dx}=\lim_{n\to\infty}\frac{\sum_{i=0}^{i=n}f^{n+1}(\frac{i}{n})}{\sum_{i=0}^{i=n}f^{n}(\frac{i}{n})}$

Do hàm f(x) liên tục$[0;1] \Rightarrow $ tồn tại $x_{0} $ sao cho$ f(x_{0})=maxf(x) ;x_{0} \in [0;1]$

Theo quy tắc ngắt bỏ VCL$\Rightarrow \frac{\int_{0}^{1}f^{n+1}(x)dx}{\int_{0}^{1}f^{n}(x)dx}=\lim_{n\to\infty}\frac{\sum_{i=0}^{i=n}f^{n+1}(\frac{i}{n})}{\sum_{i=0}^{i=n}f^{n}(\frac{i}{n})}

=\lim_{n\to\infty}\dfrac{f^{n+1}(x_{0})}{f^{n}(x_{0})}=f(x_{0})=Maxf(x)$




#601411 Tuyển tập một số bài toán Olympic SV Giải tích

Đã gửi bởi LangTu Mua Bui on 03-12-2015 - 16:57 trong Thảo luận về các kì thi, các kì kiểm tra Toán sinh viên

Câu 7 Như dùng đl Stolez nhưng vẫn có điểm gì đó cần lưu ý 
Câu 5 $u(x)\leq 1+\int_{0}^{x}\frac{\varphi '(t)u(t)dt}{\varphi (t)}$

Dễ thấy $ u(0) \leq 1$
 
$\Leftrightarrow u(x)-\varphi (x)  \leq  \int_{0}^{x} \frac{\varphi '(t)u(t)dt}{\varphi (t)}-\int_{0}^{x}\varphi '(t)dt=\int_{0}^{x}\left (\varphi '(t)( \frac{u(t)-\varphi (t)}{\varphi (t)})  \right ) dt $

Do $\varphi (t)$ đồng biến và $\varphi(0)=1 \Rightarrow \varphi (t)\geq 1 \forall t\in [0;\infty) $

$\Rightarrow u(x)-\varphi (x) \leq  \int_{0}^{x}\left (\varphi '(t)(u(x)-1)  \right ) dt=u(x)-\varphi (x)-\int_{0}^{x}\varphi '(x) $

$\Rightarrow \varphi(x)<1-\int_{0}^{x}u'(t)\varphi (t)dt ;\varphi(x)\geq 1 \forall x\in [0;\infty] \Rightarrow  u'(t)<0 $

Xét hàm số $g(x)=u(x)-\varphi (x) $

$g'(x)=u'(x)-\varphi' (x) <0 ;g(0)=u(0)-\varphi (0)<0 \Rightarrow g(x)<0 \forall x\in [0;\infty]$



#599240 Tính giá trị biểu thức $\arctan u+\arctan v+\arctan...

Đã gửi bởi LangTu Mua Bui on 20-11-2015 - 16:15 trong Phương trình - Hệ phương trình - Bất phương trình

Gọi $u,v,w$ là ba nghiệm của phương trình $x^3-10x+11=0$. Tính giá trị của biểu thức $$\arctan u+\arctan v+\arctan w$$.

Tạp chí KoMaL, Hungary.

Ta có đẳng thức quen thuộc $x=\tan{\arctan{x}}$

Biển đổi qua Ta có công thức dễ dàng  

$\Rightarrow \arctan{x}+\arctan{y}=\arctan{\tan{\left ( (\arctan{x})+(\arctan{y}) \right )}} =\arctan{\left ( \frac{\tan{(\arctan{x})}+\tan{(\arctan{y})}}{1+\tan{(\arctan{x})}\tan{(\arctan{y})}} \right )}$
 
Theo đl viet bậc cao 
$\Rightarrow \arctan{x}+\arctan{y}+\arctan{z}=\arctan{\frac{x+y+z+xyz}{1+xy+zx+yz}}$
$\left\{\begin{matrix} x+y+z=\frac{-b}{a}=0\\ xy+yz+zx=\frac{c}{a}=-10 \\zyz=\frac{d}{a}=11 \end{matrix}\right.$

$\arctan{x}+\arctan{y}+\arctan{z}=\arctan{\frac{x+y+z+xyz}{1+xy+zx+yz}}=\arctan{-\frac{11}{9}}$

 



#714054 Hạng của ánh xạ tuyến tính

Đã gửi bởi LangTu Mua Bui on 08-08-2018 - 19:39 trong Đại số tuyến tính, Hình học giải tích

Sử dụng định lý : $n=dim(imf)+dim(kerf) $ Với  $dim (imf)=rank(f)= rank (A) $với A là ma trận biểu diễn ánh xạ tuyến tính f.
Vì thế ta luôn có không gian ảnh có số chiều nhỏ hơn không gian nguồn ( n là số chiều không gian nguồn ở định lý trên ) 
Từ đây ta cần coi $rank(g) $là tập nguồn thì $rank(f.g) $là số chiều của kg tập ảnh 




#605591 câu 1 đề thi OLP giải tích 1997

Đã gửi bởi LangTu Mua Bui on 27-12-2015 - 20:29 trong Thảo luận về các kì thi, các kì kiểm tra Toán sinh viên

cho số thực A khác 0.xét hàm số $f(x)$ xác định;liên tục trên [0;$+\infty $] và thỏa mãn $\underset{x\rightarrow +\infty}{lim}f(x)=A$. tính

$\underset{n\rightarrow +\infty}{lim}\int_{0}^{1}f(nx)dx$ 

Đặt $nx=t\Rightarrow  \int_{0}^{1}f(nx)dx=\frac{\int_{0}^{n}f(n)dt}{n}$

$\Rightarrow \lim_{n\to\infty}\int_{0}^{1}f(nx)dx=\lim_{n\to\infty}\frac{\int_{0}^{n}f(n)dt}{n} (Lopitan)=\lim_{n\to\infty}f(n)=A$



#603783 CMR tồn tại 3 số $a,b,c$ sao cho $a< b< c;a+c=2b$...

Đã gửi bởi LangTu Mua Bui on 18-12-2015 - 18:14 trong Giải tích

Cho $f(x)$ liên tục trên R ;giả sử tồn tại 2 số $x_{1};x_{2}$ sao cho $f(x_{1}).f(x_{2})< 0$.

CMR tồn tại 3 số $a,b,c$ sao cho $a< b< c;a+c=2b$ và $15f(a)+2f(b)+2014f(c)=0$

----Nguyên bản BK2014---------


xét hàm g(x)=15f(x+d)+2f(x+2d)+2014f(x+3d)$
Do $f(x)$ liên tục nên$ g(x) $cũng liên tục

Do tồn tại $x_{1}$ và $x_{2} $ sao cho$ f(x_{1})f(x_{2})<0 $

$\Rightarrow \exists (a;c) ;(c;b)  sao cho f(x)<0 \forall x \in (a;c) và f(x)>0 \forall x\in (c;b) ;f(c)=0$

$\Rightarrow \alpha ;d $sao cho$ 15f( \alpha +d)+2f( \alpha +2d)+2014f( \alpha +3d)<0 $

Với đk $\alpha_{1}+d>a;\alpha+3d<c$

Tương tự$ \alpha_{2}+d ...$
 
$(\alpha_{1}).g(\alpha_{2})<0 $nên tồn tại $ \gamma \in (\alpha_{1};\alpha_{2}) $ sao cho $g(\alpha)=0$ Thỏa mãn đề bài 

 



#716595 CM: $\left \{ X\left ( \lambda _{1}...

Đã gửi bởi LangTu Mua Bui on 15-10-2018 - 15:39 trong Đại số tuyến tính, Hình học giải tích

Đặt $ u_{1}=(1,2,1),u_{2}=(2,1,0)$

Xét pt  $ \left ( u_{1}+ \lambda _{1}u_{2} \right ) +a\left ( u_{1}+ \lambda _{2}u_{2} \right )=0 (1)$ nếu tồn tại a thì chúng phụ thuộc tt.

Mặt khác $u_{1},u_{2} $là độc lập tuyến tính trong $R^{3} $ nên $(a+1)u_{1}+(\lambda+a\lambda_{2})=0 $ khi và chỉ khi hay pt (1) =0 khi và chỉ khi $ a=-1,\lambda_{1}=lambda_{2} $ trái đk giả thiết nên pt k tồn tại hay 2 vector đó thể đltt.



#714733 Chứng minh: $det(J(Y,X))= (detA)^{n}.(detB)^{n}$

Đã gửi bởi LangTu Mua Bui on 23-08-2018 - 23:25 trong Đại số tuyến tính, Hình học giải tích

Viết J(X,Y)=$\frac{\partial y_{i}}{\partial x_{i}}$ là không ổn vì dễ nhầm là phép cuộn .Để tránh nhầm lẫn nên viết lại thành $\frac{\partial y_{i}}{\partial x_{j}}$

Ở bài này nên biết đến tích tensor của 2 ma trận thì dễ hiểu hơn  $J=B^{T} \bigotimes  A$

Biến đổi từ đề bài ta được : $\Rightarrow  y_{ij}= \frac{\partial y_{ih}}{\partial x_{mn}}= \frac{\partial \left ( \sum_{j=1}^{n}\sum_{k=1}^{n}a_{ij}x_{jk}b_{kh} \right ) }{\partial x_{mn}}$

$y_{ij} $ Chỉ khác 0 khi mn trùng với jk .Suy ra ma trận Jacobi biến đổi là 1 ma trận $n^{2}\times n^{2}$

   

Đặt $c_{ij}=b_{ij} $ và  viết dạng ma trận khối 

 

                             $J=\begin{bmatrix} c_{11}A &c_{12}A&.. &c_{nn}A \\ c_{21}A & .. &...& .. \\ ... &...& ...\\ c_{n1}A & ..&.. & c_{nn}A \end{bmatrix}$

 

 

Do cách tính định thức là tổng các tính các hoán vị nên khi nhân các phần tử của $c_{ij}$ với A thì không làm thay đổi hoán vị
Gọi F là trường vector các ma trận vuông cấp n là cấp của mỗi khối , R là trường các ma trận vuông $n^{2} $ của ma trận J 
 

Điều này dẫn đến : $det J=det_{F}(det_{R} J) $

 

                                      $  =det_{F}\left (sum _{\pi \in S_{n}}(sgn \pi)c_{1\pi_{i_{1}}}c_{2\pi_{i_{2}}}...c_{n\pi_{i_{n}}}A^{n} \right )$

 

$det_{F}$ là tính định thức với dạng ma trận khối ,$det_{R}$ là tính định thức coi các ma trận thông thường bằng cách coi các  khối như phần tử vô hướng. 

 

 Do $A^{n} $là ma trận vuông cấp n                 $=det_{F}A^{n}(\left (sum _{\pi \in S_{n}}(sgn \pi)c_{1\pi_{i_{1}}}c_{2\pi_{i_{2}}}...c_{n\pi_{i_{n}}}\right )^{n}$

                                                                          $=detA^{n}detC^{n}$

                                                                          $=detA^{n}(detB^{T})^{n}$

                                                   

                                                                  $  \Rightarrow det J=(detA)^{n}(detB)^{n}$




#598220 Chứng minh Xn= (1+1/2)(1+1/4)...(1/2^n) có giới hạn.

Đã gửi bởi LangTu Mua Bui on 13-11-2015 - 23:07 trong Giải tích

$x_{n}=\prod_{k=1}^{n}(1+\frac{1}{2^{n}})=2.\left ( 1-\frac{1}{2} \right )\prod_{k=1}^{n}\left ( 1+\frac{1}{2^{k}} \right )=2(1-\frac{1}{2})(1+\frac{1}{2})(1+\frac{1}{4})..(1+\frac{1}{2^{n}})=2\left ( 1-\frac{1}{2^{2^{n}}} \right )\Rightarrow  $ Dãy số có giới hạn =2 




#599258 Chứng minh rằng với mọi n nguyên dương ta có $\dfrac{1}{2n-1} +...

Đã gửi bởi LangTu Mua Bui on 20-11-2015 - 18:56 trong Giải tích

 $f(x+1)-f(x)=f'(c) c\in (x;x+1)Roll$

Xét hàm $f(x)=\ln{x+1} \ln{x+1}-\ln{x}=\frac{1}{c} < \frac{1}{x}c\in (c;c+1)$

$ \Rightarrow \sum_{k=n}^{2n}\frac{1}{k}>\sum_{k=n}^{2n}\ln{(k+1)}-\ln{k}=\ln{2n}-\ln{n}=\ln{2}$



#714547 Chứng minh rằng các ma trận $M,N$ đồng dạng và các ma trận $P,...

Đã gửi bởi LangTu Mua Bui on 19-08-2018 - 10:40 trong Đại số tuyến tính, Hình học giải tích

Hai ma trận có cùng đa thức tối tiểu không nhất thiết phải đồng dạng với nhau, điều ấy chỉ đúng với ma trận cỡ $3\times 3$ và sai với mọi $n>3$.

 

http://www.mathcount...ic-polynomials/

Để sửa và bổ sung thêm đoạn chứng minh cùng số khối 
Hai ma trận vuông cùng cấp đồng dạng với nhau khi và chỉ khi chúng cùng dạng chuẩn tắc 
Lời bổ sung sẽ thêm sau đây




#714513 Chứng minh rằng các ma trận $M,N$ đồng dạng và các ma trận $P,...

Đã gửi bởi LangTu Mua Bui on 18-08-2018 - 14:31 trong Đại số tuyến tính, Hình học giải tích

Hai ma trận đồng dạng với nhau cho chúng có cùng đa thức cực tiểu như vậy ta có thể chéo hóa ma trận đó về dạng jordan .Đây là cách tổng quát cho mọi ma trận cho dù ma trận đó k có cơ sở để chéo hóa được.

 

 Các ma trận đường chéo $ A=diag(a_{ii}) $ có đặc điểm thuận tiện khi lũy thừa vì thế một đa thức P(x) thì  P(A) là một ma trận đường chéo có dạng $diag(P(a_{ii})) $ nó đúng cho cả các ma trận khối .
.
Ta  có tính chất nếu đa thức P(A)=0 thì nó là chia hết cho $g_{A} $ đa thức cực tiểu của ma trận A  
Xét P(N) là đa thức tối thiểu thì $diag(P(A+B) ,P(A-B) )$ Vì thế P(N)=0 khi chỉ khi P(A+B)=0 ,P(A-B) =0 tức P(N) là bội chung của của 2 đa thức cực tiểu A+B và A-B. .
Do  $g_{N} $là đa thức cực tiểu vì vậy là đa thức có bậc bé nhất suy ra nó là  bội chung nhỏ nhất của 2 đa thức cực tiểu $g_{A+B} $và $g_{A-B} $ 

Bây h ta sẽ đi chứng minh ma trận M cũng có đa thức cực tiểu là bội chung nhỏ nhất của $g_{A+B}$ và $g_{A-B}$
Xét thấy 

$\begin{pmatrix} A & B\\ B&A \end{pmatrix}=\frac{1}{2}\begin{pmatrix} (A+B)+(A-B) & (A+B)-(A-B) \\ (A+B)-(A-B)& (A+B)+(A-B) \end{pmatrix}$

$\begin{pmatrix} A &B \\ B&A \end{pmatrix}^{2}=\frac{1}{2}\begin{pmatrix} (A+B)^{2}+(A-B)^{2} & (A+B)^{2}-(A-B)^{2} \\ (A+B)^{2}-(A-B)^{2}& (A+B)^{2}+(A-B)^{2} \end{pmatrix}$
.. ..
$P(M)=\frac{1}{2}\begin{pmatrix} P(A+B)+P(A-B) & P(A+B)-P(A-B)\\ P(A+B)-P(A-B)& P(A+B)+P(A-B) \end{pmatrix}$
 

Tương tự nếu $P(M)=0 $thì $P(A+B)+P(A-B)=0$ và $P(A+B)-P(A-B)=0$ điều đó có nghĩa là P(M)=0 khi và chỉ khi$ P(A+B) $và $P(A-B) $đều bằng 0.
Dẫn tới các đa thức thỏa mãn$ g(M)=0$ thì chúng là bội chung của các đa thức cực tiểu $(A+B) $và $(A-B) $

Nếu là đa thức cực tiểu số bậc nhỏ nhất thì nó là duy nhất và là bội chung nhỏ nhất của 2 đa thức cực tiểu của 2 ma trận $(A+B) $và $(A-B) .$

Vậy ta đã chứng minh 2 ma trận M và N cùng đa thức cực tiểu vì thế chúng đồng dạng với nhau.




#714549 Chứng minh rằng các ma trận $M,N$ đồng dạng và các ma trận $P,...

Đã gửi bởi LangTu Mua Bui on 19-08-2018 - 11:22 trong Đại số tuyến tính, Hình học giải tích

Hai ma trận đồng dạng với nhau khi chúng cùng dạng chuẩn tắc như vậy ta có thể chéo hóa ma trận đó về dạng jordan .Đây là cách tổng quát cho mọi ma trận cho dù ma trận đó k có cơ sở để chéo hóa được.

Đầu tiên đi chứng minh chúng cùng đa thức đặc trưng :

 

 Các ma trận đường chéo $ A=diag(a_{ii}) $ có đặc điểm thuận tiện khi lũy thừa vì thế một đa thức P(x) thì  P(A) là một ma trận đường chéo có dạng $diag(P(a_{ii})) $ nó đúng cho cả các ma trận khối .
.
Ta  có tính chất nếu đa thức P(A)=0 thì nó là chia hết cho $g_{A} $ đa thức cực tiểu của ma trận A  
Xét P(N) là đa thức tối thiểu thì $diag(P(A+B) ,P(A-B) )$ Vì thế P(N)=0 khi chỉ khi P(A+B)=0 ,P(A-B) =0 tức P(N) là bội chung của của 2 đa thức cực tiểu A+B và A-B. .
Do  $g_{N} $là đa thức cực tiểu vì vậy là đa thức có bậc bé nhất suy ra nó là  bội chung nhỏ nhất của 2 đa thức cực tiểu $g_{A+B} $và $g_{A-B} $ 

Bây h ta sẽ đi chứng minh ma trận M cũng có đa thức cực tiểu là bội chung nhỏ nhất của $g_{A+B}$ và $g_{A-B}$
Xét thấy 

$\begin{pmatrix} A & B\\ B&A \end{pmatrix}=\frac{1}{2}\begin{pmatrix} (A+B)+(A-B) & (A+B)-(A-B) \\ (A+B)-(A-B)& (A+B)+(A-B) \end{pmatrix}$

$\begin{pmatrix} A &B \\ B&A \end{pmatrix}^{2}=\frac{1}{2}\begin{pmatrix} (A+B)^{2}+(A-B)^{2} & (A+B)^{2}-(A-B)^{2} \\ (A+B)^{2}-(A-B)^{2}& (A+B)^{2}+(A-B)^{2} \end{pmatrix}$
.. ..
$P(M)=\frac{1}{2}\begin{pmatrix} P(A+B)+P(A-B) & P(A+B)-P(A-B)\\ P(A+B)-P(A-B)& P(A+B)+P(A-B) \end{pmatrix}$
 

Tương tự nếu $P(M)=0 $thì $P(A+B)+P(A-B)=0$ và $P(A+B)-P(A-B)=0$ điều đó có nghĩa là P(M)=0 khi và chỉ khi$ P(A+B) $và $P(A-B) $đều bằng 0.
Dẫn tới các đa thức thỏa mãn$ g(M)=0$ thì chúng là bội chung của các đa thức cực tiểu $(A+B) $và $(A-B) $

Nếu là đa thức cực tiểu số bậc nhỏ nhất thì nó là duy nhất và là bội chung nhỏ nhất của 2 đa thức cực tiểu của 2 ma trận $(A+B) $và $(A-B) .$

Tiếp theo chứng minh chúng cùng mỗi khối liên kết tương ứng bằng nhau 

Tức thỏa mãn định lý số chiều gọi S_{ik} là số khối của đa thức liên kết 
Gọi là g là đa thức tối thiểu của ma trận M $g=g^{p_{1}}_{1}...g^{p_{r}}_{n} $
$S_{ik}=\frac{rank g^{k-1}_{i} -2rank g^{k}_{i}+rank g^{k+1}_{i} }{deg g_{i}} $
 

Với ma trận N ta có $rankg_{i}(A)=\begin{pmatrix} g_{i}(A+B) &0 \\ 0 & g_{i}(A-B) \end{pmatrix}$ suy ra $rank g_{i}N)=rank(g_{i}(A+B))+rank g_{i}(A-B) $ 
 

Với ma trận M nhờ cộng theo hàng ta được $ rank g_{i}(M)$=rank $\begin{pmatrix} g_{i}(A+B)+g_{i}(A-B) & g_{i}(A+B)-g_{i}(A-B)\\ g_{i}(A+B)-g_{i}(A-B) & g_{i}(A+B)+g_{i}(A-B) \end{pmatrix} =$rank $\begin{pmatrix} g_{i}{A+B} &g_{i}{A+B} \\ -g_{i}{A-B} &g_{i}(A-B) \end{pmatrix}$
Ta thấy mỗi hạng của ma trận của khối thứ 1 khi biến đổi sơ cấp với các hạng của khối thứ 2 đều không thể bằng 0 điều này chứng tỏ  hạng của ma trận M bằng hạng $rank g_{i}(A+B) +rank g_{i}(A-B) $ 
Điều này chứng tỏ số khối liên kết của 2 ma trận tương tứng bằng nhau .

Vậy ta đã chứng minh 2 ma trận M và N cùng đa thức cực tiểu vì thế chúng đồng dạng với nhau.




#598961 Chứng minh phương trình có nghiệm

Đã gửi bởi LangTu Mua Bui on 18-11-2015 - 17:12 trong Giải tích

xét hàm số $f(x)=\frac{ax^{4}}{4}+bx^{2}+2cx$

Ta có $f(0)=0;f(2)=4a+4b+4c=4(a+b+c)=0 $ Theo đl Roll 

$\Rightarrow f(0)=f(2)\Rightarrow  \exists c \in (0;2) f'(c)=0$

$ \Leftrightarrow pt ax^{3}+2bx+2c=0$có nghiệm$ (0;2)$



#599311 Cho $k\in \mathbb{N}^*$. Tìm $$L=\lim_{n...

Đã gửi bởi LangTu Mua Bui on 20-11-2015 - 22:03 trong Giải tích

$\int_{0}^{1}\frac{\ln{(1+x^{n+k})}}{\ln{(1+x^{n})}}=\int_{0}^{1-\varepsilon }\frac{\ln{(1+x^{n+k})}}{\ln{(1+x^{n})}}+\int_{1-\varepsilon }^{1}\frac{\ln{(1+x^{n+k})}}{\ln{(1+x^{n})}} =I_{1}+I_{2} $

$I_{2}=\varepsilon \frac{\ln{(1+{c}^{n+k})}}{\ln{(1+{c}^{n})}}=0 I_{1}=\int_{0}^{1-\varepsilon }\frac{x^{n+k}}{x^{n}}=\int_{0}^{1-\varepsilon }x^{k}$

$=\lim_{\varepsilon \to\ 0}\frac{x^{k+1}}{k+1}|^{1-\varepsilon }_{0} =\frac{1}{k+1}$